वुडबरी आव्यूह समरूपता: Difference between revisions

From Vigyanwiki
No edit summary
No edit summary
Line 1: Line 1:


{{Short description|Theorem of matrix ranks}}
{{Short description|Theorem of matrix ranks}}
गणित में (विशेष रूप से रैखिक बीजगणित), वुडबरी आव्यूह समरूपता, जिसका नाम मैक्स ए। वुडबरी के नाम पर रखा गया है,<ref>Max A. Woodbury, ''Inverting modified matrices'', Memorandum Rept. 42, Statistical Research Group, Princeton University, Princeton, NJ, 1950, 4pp {{MR|38136}}</ref><ref>Max A. Woodbury, ''The Stability of Out-Input Matrices''. Chicago, Ill., 1949. 5 pp. {{MR|32564}}</ref> जो यह कहते है कि कुछ [[मैट्रिक्स (गणित)|आव्यूह (गणित)]] के पद-k सुधार के व्युत्क्रम की गणना मूल आव्यूह के व्युत्क्रम में पद-k सुधार करके की जा सकती है। इस सूत्र के वैकल्पिक नाम 'आव्यूह व्युत्क्रमता लेम्मा', 'शर्मन-मॉरिसन-वुडबरी सूत्र' या मात्र 'वुडबरी सूत्र' हैं। यद्यपि, वुडबरी रिपोर्ट से पहले यह समरूपता कई लेखों में छपी थी।<ref name="guttman">{{cite journal
गणित में (विशेष रूप से रैखिक बीजगणित), '''वुडबरी आव्यूह समरूपता''', जिसका नाम मैक्स ए. वुडबरी के नाम पर रखा गया है,<ref>Max A. Woodbury, ''Inverting modified matrices'', Memorandum Rept. 42, Statistical Research Group, Princeton University, Princeton, NJ, 1950, 4pp {{MR|38136}}</ref><ref>Max A. Woodbury, ''The Stability of Out-Input Matrices''. Chicago, Ill., 1949. 5 pp. {{MR|32564}}</ref> जो यह कहते है कि कुछ [[मैट्रिक्स (गणित)|आव्यूह (गणित)]] के पद-k सुधार के व्युत्क्रम की गणना मूल आव्यूह के व्युत्क्रम में पद-k सुधार करके की जा सकती है। अतः इस सूत्र के वैकल्पिक नाम '<nowiki/>'''आव्यूह व्युत्क्रमता लेम्मा', 'शर्मन-मॉरिसन-वुडबरी सूत्र'<nowiki/>''' या '''मात्र 'वुडबरी सूत्र'''' हैं। यद्यपि, वुडबरी रिपोर्ट से पहले यह समरूपता कई लेखों में छपी थी।<ref name="guttman">{{cite journal
  |first=Louis |last=Guttmann
  |first=Louis |last=Guttmann
  |title=Enlargement methods for computing the inverse matrix
  |title=Enlargement methods for computing the inverse matrix
Line 15: Line 15:
}}</ref>
}}</ref>


वुडबरी आव्यूह समरूपता<ref name="higham">{{Cite book | last1=Higham | first1=Nicholas | author1-link=Nicholas Higham | title=संख्यात्मक एल्गोरिदम की सटीकता और स्थिरता| url=https://archive.org/details/accuracystabilit00high_878 | url-access=limited | publisher=[[Society for Industrial and Applied Mathematics|SIAM]] | edition=2nd | isbn=978-0-89871-521-7 | year=2002 | page=[https://archive.org/details/accuracystabilit00high_878/page/n288 258] |mr=1927606 }}
इस प्रकार से वुडबरी आव्यूह समरूपता<ref name="higham">{{Cite book | last1=Higham | first1=Nicholas | author1-link=Nicholas Higham | title=संख्यात्मक एल्गोरिदम की सटीकता और स्थिरता| url=https://archive.org/details/accuracystabilit00high_878 | url-access=limited | publisher=[[Society for Industrial and Applied Mathematics|SIAM]] | edition=2nd | isbn=978-0-89871-521-7 | year=2002 | page=[https://archive.org/details/accuracystabilit00high_878/page/n288 258] |mr=1927606 }}
</ref>
</ref>
:<math> \left(A + UCV \right)^{-1} = A^{-1} - A^{-1}U \left(C^{-1} + VA^{-1}U \right)^{-1} VA^{-1} </math>
:<math> \left(A + UCV \right)^{-1} = A^{-1} - A^{-1}U \left(C^{-1} + VA^{-1}U \right)^{-1} VA^{-1} </math>
Line 22: Line 22:
जबकि समरूपता मुख्य रूप से आव्यूह पर उपयोग की जाती है, यह सामान्य वलय (गणित) या [[एब-श्रेणी|Ab-श्रेणी]] में होती है।
जबकि समरूपता मुख्य रूप से आव्यूह पर उपयोग की जाती है, यह सामान्य वलय (गणित) या [[एब-श्रेणी|Ab-श्रेणी]] में होती है।


वुडबरी आव्यूह समरूपता व्युत्क्रमों की तुच्छ गणना और रैखिक समीकरणों के हल की अनुमति देती है। यद्यपि, सूत्र की संख्यात्मक स्थिरता के विषय में बहुत कम सूचना है। इसकी त्रुटि सीमा के संबंध में कोई प्रकाशित परिणाम नहीं हैं। उपाख्यानात्मक प्रमाण <ref>
इस प्रकार से वुडबरी आव्यूह समरूपता व्युत्क्रमों की तुच्छ गणना और रैखिक समीकरणों के हल की अनुमति देती है। यद्यपि, सूत्र की संख्यात्मक स्थिरता के विषय में बहुत कम सूचना है। अतः इसकी त्रुटि सीमा के संबंध में कोई प्रकाशित परिणाम नहीं हैं। उपाख्यानात्मक प्रमाण <ref>
{{cite web  
{{cite web  
| url = https://mathoverflow.net/questions/80340/special-considerations-when-using-the-woodbury-matrix-identity-numerically
| url = https://mathoverflow.net/questions/80340/special-considerations-when-using-the-woodbury-matrix-identity-numerically
Line 31: Line 31:


==चर्चा==
==चर्चा==
इस परिणाम को सिद्ध करने के लिए, हम सरल परिणाम को सिद्ध करके प्रारम्भ करेंगे। A और C को समरूपता आव्यूह I के साथ प्रतिस्थापित करने पर, हमें और समरूपता प्राप्त होती है जो थोड़ी सरल है:
इस परिणाम को सिद्ध करने के लिए, हम सरल परिणाम को सिद्ध करके प्रारम्भ करेंगे। इस प्रकार से A और C को समरूपता आव्यूह I के साथ प्रतिस्थापित करने पर, हमें और समरूपता प्राप्त होती है जो थोड़ी सरल है:
:<math> \left(I + UV \right)^{-1} = I - U \left(I + VU \right)^{-1} V. </math>
:<math> \left(I + UV \right)^{-1} = I - U \left(I + VU \right)^{-1} V. </math>
इस घटी हुई समरूपता से मूल समीकरण को पुनः प्राप्त करने के लिए, समुच्चय <math>U = A^{-1}X</math> और <math>V = CY</math> है।
अतः इस घटी हुई समरूपता से मूल समीकरण को पुनः प्राप्त करने के लिए, समुच्चय <math>U = A^{-1}X</math> और <math>V = CY</math> है।


इस समरूपता को ही दो सरल समरूपताों के संयोजन के रूप में देखा जा सकता है। हम पहली समरूपता
इस समरूपता को ही दो सरल समरूपताों के संयोजन के रूप में देखा जा सकता है। इस प्रकार से हम पहली समरूपता
: <math> I = (I + P)^{-1}(I + P) = (I + P)^{-1} + (I + P)^{-1}P</math>
: <math> I = (I + P)^{-1}(I + P) = (I + P)^{-1} + (I + P)^{-1}P</math>
से प्राप्त करते हैं, इस प्रकार,
से प्राप्त करते हैं, इस प्रकार,
Line 41: Line 41:
और इसी प्रकार
और इसी प्रकार
: <math> (I + P)^{-1} = I - P (I + P)^{-1}.</math>
: <math> (I + P)^{-1} = I - P (I + P)^{-1}.</math>
दूसरी समरूपता तथाकथित पुश-थ्रू समरूपता<ref name="HS"/>
दूसरी समरूपता तथाकथित '''पुश-थ्रू समरूपता'''<ref name="HS"/>


<math> (I + UV)^{-1} U = U (I + VU)^{-1} </math>
<math> (I + UV)^{-1} U = U (I + VU)^{-1} </math>
Line 59: Line 59:
जब <math>V, U</math> सदिश होते हैं, तो समरूपता शर्मन-मॉरिसन सूत्र तक कम हो जाती है।
जब <math>V, U</math> सदिश होते हैं, तो समरूपता शर्मन-मॉरिसन सूत्र तक कम हो जाती है।


अदिश स्थिति में, घटा हुआ संस्करण मात्र
इस प्रकार से अदिश स्थिति में, घटा हुआ संस्करण मात्र
: <math>\frac{1}{1 + uv} = 1 - \frac{uv}{1 + uv}</math> है।
: <math>\frac{1}{1 + uv} = 1 - \frac{uv}{1 + uv}</math> है।
==== योग का व्युत्क्रम ====
==== योग का व्युत्क्रम ====


यदि n = k और U = V = I<sub>''n''</sub> तो, समरूपता आव्यूह है
यदि n = k और U = V = I<sub>''n''</sub> तो, समरूपता आव्यूह


:<math>
:<math>
\begin{align}
\begin{align}
\left({A} + {B}\right)^{-1} &= A^{-1} - A^{-1} (B^{-1} + A^{-1})^{-1} A^{-1}\\
\left({A} + {B}\right)^{-1} &= A^{-1} - A^{-1} (B^{-1} + A^{-1})^{-1} A^{-1}\\
&= {A}^{-1} - {A}^{-1}\left({A}{B}^{-1} + {I}\right)^{-1}.
&= {A}^{-1} - {A}^{-1}\left({A}{B}^{-1} + {I}\right)^{-1}
\end{align}
\end{align}
</math>
</math> है।
उपरोक्त समीकरण के सबसे दाईं ओर के पदों के विलय को जारी रखने से हुआ की समरूपता होती है
उपरोक्त समीकरण के सबसे दाईं ओर के पदों के एक संविलय को जारी रखने से हुआ की समरूपता
:<math>\left({A} + {B}\right)^{-1} = {A}^{-1} - \left({A} + {A}{B}^{-1}{A}\right)^{-1}.</math>
:<math>\left({A} + {B}\right)^{-1} = {A}^{-1} - \left({A} + {A}{B}^{-1}{A}\right)^{-1}</math> प्राप्त होती है।
इसी समरूपता का और उपयोगी रूप है
इस प्रकार से समान समरूपता का एक अन्य उपयोगी रूप
:<math>\left({A} - {B}\right)^{-1} = {A}^{-1} + {A}^{-1}{B}\left({A} - {B}\right)^{-1},</math>
:<math>\left({A} - {B}\right)^{-1} = {A}^{-1} + {A}^{-1}{B}\left({A} - {B}\right)^{-1},</math>
जो, उपरोक्त के विपरीत, भले ही मान्य हो <math>B</math> एकवचन है, और इसमें पुनरावर्ती संरचना है जो उत्पन्न करती है
है, जो उपरोक्त के विपरीत, <math>B</math> एकल होने पर भी मान्य है, और इसमें एक पुनरावर्ती संरचना है जो <math>A^{-1}B</math> का [[वर्णक्रमीय त्रिज्या]] एक से कम होने पर
:<math>\left({A} - {B}\right)^{-1} = \sum_{k=0}^{\infty} \left({A}^{-1}{B}\right)^k{A}^{-1}</math>
:<math>\left({A} - {B}\right)^{-1} = \sum_{k=0}^{\infty} \left({A}^{-1}{B}\right)^k{A}^{-1}</math>
यदि की [[वर्णक्रमीय त्रिज्या]] <math>A^{-1}B</math> से कम है। अर्थात यदि उपरोक्त योग एकत्रित हो जाए तो बराबर हो जाता है <math>(A-B)^{-1}</math>
उत्पन्न करती है। अर्थात्, यदि उपरोक्त योग अभिसरित होता है तो यह <math>(A-B)^{-1}</math> के बराबर होता है।


इस फॉर्म का उपयोग गड़बड़ी वाले विस्तारों में किया जा सकता है जहां बी ए का गड़बड़ी है।
अतः इस रूप का उपयोग त्रुटि वाले विस्तारों में किया जा सकता है जहां B A की त्रुटि है।


=== विविधताएँ ===
=== विविधताएँ ===
Line 90: Line 90:
   A^{-1} - A^{-1}UB\left(B+BVA^{-1}UB\right)^{-1}BVA^{-1}
   A^{-1} - A^{-1}UB\left(B+BVA^{-1}UB\right)^{-1}BVA^{-1}
</math>
</math>
ए और बी + बीवीए प्रदान किया गया<sup>−1</sup>B एकवचन नहीं हैं। अक्षर की गैर विलक्षणता के लिए आवश्यक है कि बी<sup>−1</sup> अस्तित्व में है क्योंकि यह बराबर है {{nowrap|''B''(''I'' + ''VA''<sup>−1</sup>''UB'')}} और बाद वाले का पद बी के पद से अधिक नहीं हो सकता।<ref name=HS>{{cite journal | last1 = Henderson | first1 = H. V. | last2 = Searle | first2 = S. R. | year = 1981 | title = आव्यूहों के योग का व्युत्क्रम निकालने पर| url = http://ecommons.cornell.edu/bitstream/1813/32749/1/BU-647-M.pdf| journal = SIAM Review | volume = 23 | issue = 1 | pages = 53–60 | doi = 10.1137/1023004 | jstor = 2029838 | hdl = 1813/32749 | hdl-access = free }}</ref>
प्रदान किया गया है कि A और B + ''BVA''<sup>−1</sup>UB एकल नहीं हैं। इस प्रकार से उत्तरार्द्ध की गैर-विलक्षणता के लिए आवश्यक है कि B<sup>−1</sup> अस्तित्व में हो क्योंकि यह {{nowrap|''B''(''I'' + ''VA''<sup>−1</sup>''UB'')}} के बराबर है और बाद वाले की पद B की पद से अधिक नहीं हो सकती है।<ref name=HS>{{cite journal | last1 = Henderson | first1 = H. V. | last2 = Searle | first2 = S. R. | year = 1981 | title = आव्यूहों के योग का व्युत्क्रम निकालने पर| url = http://ecommons.cornell.edu/bitstream/1813/32749/1/BU-647-M.pdf| journal = SIAM Review | volume = 23 | issue = 1 | pages = 53–60 | doi = 10.1137/1023004 | jstor = 2029838 | hdl = 1813/32749 | hdl-access = free }}</ref>
चूँकि B व्युत्क्रमणीय है, दाहिनी ओर कोष्ठक में व्युत्क्रमित मात्रा को दर्शाने वाले दो B पदों को प्रतिस्थापित किया जा सकता है {{nowrap|(''B''<sup>−1</sup>)<sup>−1</sup>,}} जिसके परिणामस्वरूप मूल वुडबरी समरूपता प्राप्त होती है।


जब B एकवचन हो और संभवतः गैर-वर्ग भी हो, तो इसके लिए भिन्नता:<ref name=HS/>
चूँकि B व्युत्क्रमणीय है, दाहिनी ओर कोष्ठक में व्युत्क्रमित मात्रा को दर्शाने वाले दो B पदों को {{nowrap|(''B''<sup>−1</sup>)<sup>−1</sup>}} से प्रतिस्थापित किया जा सकता है, जिसके परिणामस्वरूप मूल वुडबरी समरूपता प्राप्त होती है।
 
इस प्रकार से जब B एकल हो और संभवतः गैर-वर्ग भी हो, तो इसके लिए भिन्नता:<ref name="HS" />


:<math>(A + UBV)^{-1} = A^{-1} - A^{-1}U(I + BVA^{-1}U)^{-1}BVA^{-1}.</math>
:<math>(A + UBV)^{-1} = A^{-1} - A^{-1}U(I + BVA^{-1}U)^{-1}BVA^{-1}.</math>
कुछ मामलों के लिए सूत्र भी मौजूद हैं जिनमें A एकवचन है।<ref>Kurt S. Riedel, "A Sherman–Morrison–Woodbury Identity for Rank Augmenting Matrices with Application to Centering", ''SIAM Journal on Matrix Analysis and Applications'', 13 (1992)659-662, {{doi|10.1137/0613040}} [http://math.nyu.edu/mfdd/riedel/ranksiam.ps preprint] {{MR|1152773}}</ref>
अतः कुछ स्थितियों के लिए सूत्र भी स्थित हैं जिनमें A एकल है।<ref>Kurt S. Riedel, "A Sherman–Morrison–Woodbury Identity for Rank Augmenting Matrices with Application to Centering", ''SIAM Journal on Matrix Analysis and Applications'', 13 (1992)659-662, {{doi|10.1137/0613040}} [http://math.nyu.edu/mfdd/riedel/ranksiam.ps preprint] {{MR|1152773}}</ref>
==== सकारात्मक अर्धनिश्चित आव्यूह के साथ छद्म व्युत्क्रम ====
==== धनात्मक अर्धनिश्चित आव्यूह के साथ छद्म व्युत्क्रम ====


सामान्य तौर पर वुडबरी की समरूपता मान्य नहीं है यदि या अधिक व्युत्क्रमों को मूर-पेनरोज़ व्युत्क्रम|(मूर-पेनरोज़) स्यूडो व्युत्क्रम द्वारा प्रतिस्थापित किया जाता है। यद्यपि, यदि <math>A</math> और <math>C</math> सकारात्मक अर्धनिश्चित आव्यूह हैं, और <math>V = U^\mathrm H</math> (इसका तात्पर्य यह है कि <math>A + UCV</math> स्वयं सकारात्मक अर्धनिश्चित है), तो निम्न सूत्र सामान्यीकरण प्रदान करता है:<ref>{{cite book |last1=Bernstein |first1=Dennis S. |title=Scalar, Vector, and Matrix Mathematics: Theory, Facts, and Formulas |date=2018 |publisher=Princeton University Press |location=Princeton |isbn=9780691151205 |page=638 |edition=Revised and expanded}}</ref><ref>{{cite book |last1=Schott |first1=James R. |title=सांख्यिकी के लिए मैट्रिक्स विश्लेषण|date=2017 |publisher=John Wiley & Sons, Inc. |location=Hoboken, New Jersey |isbn=9781119092483 |page=219 |edition=Third}}</ref>
इस प्रकार से सामान्यतः वुडबरी की समरूपता मान्य नहीं है यदि या अधिक व्युत्क्रमों को मूर-पेनरोज़ व्युत्क्रम या (मूर-पेनरोज़) छद्म व्युत्क्रम द्वारा प्रतिस्थापित किया जाता है। यद्यपि, यदि <math>A</math> और <math>C</math> धनात्मक अर्धनिश्चित आव्यूह हैं, और <math>V = U^\mathrm H</math> (इसका तात्पर्य यह है कि <math>A + UCV</math> स्वयं धनात्मक अर्धनिश्चित है), तो निम्न सूत्र सामान्यीकरण प्रदान करता है:<ref>{{cite book |last1=Bernstein |first1=Dennis S. |title=Scalar, Vector, and Matrix Mathematics: Theory, Facts, and Formulas |date=2018 |publisher=Princeton University Press |location=Princeton |isbn=9780691151205 |page=638 |edition=Revised and expanded}}</ref><ref>{{cite book |last1=Schott |first1=James R. |title=सांख्यिकी के लिए मैट्रिक्स विश्लेषण|date=2017 |publisher=John Wiley & Sons, Inc. |location=Hoboken, New Jersey |isbn=9781119092483 |page=219 |edition=Third}}</ref>
:<math>
:<math>
\begin{align}
\begin{align}
Line 109: Line 110:
\end{align}
\end{align}
</math>
</math>
कहाँ <math>A + UCU^\mathrm H</math> के रूप में लिखा जा सकता है <math>XX^\mathrm H + YY^\mathrm H</math> क्योंकि कोई भी सकारात्मक अर्धनिश्चित आव्यूह बराबर है <math>MM^\mathrm H</math> कुछ के लिए <math>M</math>
जहाँ <math>A + UCU^\mathrm H</math> को <math>XX^\mathrm H + YY^\mathrm H</math> के रूप में लिखा जा सकता है क्योंकि कोई भी धनात्मक अर्धनिश्चित आव्यूह कुछ <math>M</math> के लिए <math>MM^\mathrm H</math> के बराबर है।


== व्युत्पत्तियाँ ==
== व्युत्पत्तियाँ ==


===प्रत्यक्ष प्रमाण ===
===प्रत्यक्ष प्रमाण ===
उसकी जांच करके सूत्र को सिद्ध किया जा सकता है <math>(A + UCV)</math> कई बार वुडबरी समरूपता के दाईं ओर इसका कथित उलटा समरूपता आव्यूह देता है:
इस प्रकार से सूत्र को यह जांच कर सिद्ध किया जा सकता है कि वुडबरी समरूपता के दाईं ओर <math>(A + UCV)</math> गुना इसके कथित व्युत्क्रम से समरूपता आव्यूह मिलता है:


: <math>\begin{align}
: <math>\begin{align}
Line 127: Line 128:
=== वैकल्पिक प्रमाण ===
=== वैकल्पिक प्रमाण ===


{{collapse top|title=Algebraic proof }}
{{collapse top|title=बीजगणितीय प्रमाण }}
पहले इन उपयोगी पहचानों पर विचार करें,
पहले इन उपयोगी समरूपताओं पर विचार करें,
: <math>\begin{align}
: <math>\begin{align}
               U + UCV A^{-1} U &=  
               U + UCV A^{-1} U &=  
Line 144: Line 145:
{{collapse bottom}}
{{collapse bottom}}


{{collapse top|title=Derivation via blockwise elimination}}
{{collapse top|title=ब्लॉकवार उन्मूलन के माध्यम से व्युत्पत्ति}}
वुडबरी मैट्रिक्स पहचान प्राप्त करना निम्नलिखित ब्लॉक मैट्रिक्स व्युत्क्रम समस्या को हल करके आसानी से किया जाता है
वुडबरी आव्यूह समरूपता प्राप्त करना निम्नलिखित ब्लॉक आव्यूह व्युत्क्रम समस्या
:<math>
:<math>
   \begin{bmatrix} A & U \\ V & -C^{-1} \end{bmatrix}\begin{bmatrix} X \\ Y \end{bmatrix} = \begin{bmatrix} I \\ 0 \end{bmatrix}.
   \begin{bmatrix} A & U \\ V & -C^{-1} \end{bmatrix}\begin{bmatrix} X \\ Y \end{bmatrix} = \begin{bmatrix} I \\ 0 \end{bmatrix}
</math>
</math>
विस्तार करते हुए, हम देख सकते हैं कि उपरोक्त कम हो जाता है
को हल करके सरलता से किया जाता है।
विस्तार करते हुए, हम देख सकते हैं कि उपरोक्त


:<math>\begin{cases} AX + UY = I \\ VX - C^{-1}Y = 0\end{cases}</math>
:<math>\begin{cases} AX + UY = I \\ VX - C^{-1}Y = 0\end{cases}</math>
जो के बराबर है <math>(A + UCV)X = I</math>. पहले समीकरण को हटाकर, हम उसे पाते हैं <math>X = A^{-1}(I - UY)</math>, जिसे खोजने के लिए दूसरे में प्रतिस्थापित किया जा सकता है <math>VA^{-1}(I - UY) = C^{-1}Y</math>. विस्तार और पुनर्व्यवस्थित करना, हमारे पास है <math>VA^{-1} = \left(C^{-1} + VA^{-1}U\right)Y</math>, या <math>\left(C^{-1} + VA^{-1}U\right)^{-1}VA^{-1} = Y</math>. अंत में, हम अपने में स्थानापन्न करते हैं <math>AX + UY = I</math>, और हमारे पास है <math>AX + U\left(C^{-1} + VA^{-1}U\right)^{-1}VA^{-1} = I</math>. इस प्रकार,
तक कम हो जाता है जो <math>(A + UCV)X = I</math> के बराबर है। पहले समीकरण को हटाकर, हम उस <math>X = A^{-1}(I - UY)</math>को पाते हैं, जिसे <math>VA^{-1}(I - UY) = C^{-1}Y</math>खोजने के लिए दूसरे में प्रतिस्थापित किया जा सकता है। विस्तार और पुनर्व्यवस्थित करते हुए, हमारे निकट <math>VA^{-1} = \left(C^{-1} + VA^{-1}U\right)Y</math>, या <math>\left(C^{-1} + VA^{-1}U\right)^{-1}VA^{-1} = Y</math>है। अंत में, हम अपने <math>AX + UY = I</math>में स्थानापन्न करते हैं, और हमारे निकट <math>AX + U\left(C^{-1} + VA^{-1}U\right)^{-1}VA^{-1} = I</math>होता है। इस प्रकार,
:<math>(A + UCV)^{-1} = X = A^{-1} - A^{-1}U\left(C^{-1} + VA^{-1}U\right)^{-1}VA^{-1}.</math>
:<math>(A + UCV)^{-1} = X = A^{-1} - A^{-1}U\left(C^{-1} + VA^{-1}U\right)^{-1}VA^{-1}.</math>
हमने वुडबरी मैट्रिक्स पहचान प्राप्त की है।
हमने वुडबरी आव्यूह समरूपता प्राप्त की है।
{{collapse bottom}}
{{collapse bottom}}


{{collapse top|title=Derivation from LDU decomposition}}
{{collapse top|title=एलडीयू अपघटन से व्युत्पत्ति}}
हम मैट्रिक्स से शुरू करते हैं
हम आव्यूह
:<math>\begin{bmatrix} A & U \\ V & C \end{bmatrix}</math>
:<math>\begin{bmatrix} A & U \\ V & C \end{bmatrix}</math>
के अंतर्गत प्रविष्टि को हटाने पर (यह देखते हुए कि ए उलटा है) हमें मिलता है
से प्रारम्भ करते हैं
A के अंतर्गत प्रविष्टि को हटाने पर (यह देखते हुए कि A व्युत्क्रम है) हमें


:<math>\begin{bmatrix} I & 0 \\ -VA^{-1} & I \end{bmatrix}  
:<math>\begin{bmatrix} I & 0 \\ -VA^{-1} & I \end{bmatrix}  
\begin{bmatrix} A & U \\ V & C \end{bmatrix} = \begin{bmatrix} A & U \\ 0 & C - VA^{-1}U \end{bmatrix}
\begin{bmatrix} A & U \\ V & C \end{bmatrix} = \begin{bmatrix} A & U \\ 0 & C - VA^{-1}U \end{bmatrix}
</math>
</math> मिलता है
इसी तरह, C से ऊपर की प्रविष्टि को हटा देना
इसी प्रकार, C के ऊपर की प्रविष्टि को हटाने से


:<math>\begin{bmatrix} A & U \\ V & C \end{bmatrix} \begin{bmatrix} I & -A^{-1}U \\ 0 & I \end{bmatrix}  
:<math>\begin{bmatrix} A & U \\ V & C \end{bmatrix} \begin{bmatrix} I & -A^{-1}U \\ 0 & I \end{bmatrix}  
= \begin{bmatrix} A & 0 \\ V & C-VA^{-1}U \end{bmatrix}
= \begin{bmatrix} A & 0 \\ V & C-VA^{-1}U \end{bmatrix}
</math>
</math> मिलता है
अब उपरोक्त दोनों को मिलाने पर हमें प्राप्त होता है
अब उपरोक्त दोनों को मिलाकर हमें


:<math>
:<math>
   \begin{bmatrix} I & 0 \\ -VA^{-1} & I \end{bmatrix} \begin{bmatrix} A & U \\ V & C \end{bmatrix}\begin{bmatrix} I & -A^{-1}U \\ 0 & I \end{bmatrix} =
   \begin{bmatrix} I & 0 \\ -VA^{-1} & I \end{bmatrix} \begin{bmatrix} A & U \\ V & C \end{bmatrix}\begin{bmatrix} I & -A^{-1}U \\ 0 & I \end{bmatrix} =
   \begin{bmatrix} A & 0 \\ 0 & C - VA^{-1}U \end{bmatrix}
   \begin{bmatrix} A & 0 \\ 0 & C - VA^{-1}U \end{bmatrix}
</math>
</math> प्राप्त होता है
दायीं ओर जाने से लाभ मिलता है
दाईं ओर जाने पर


:<math>\begin{bmatrix} A & U \\ V & C \end{bmatrix} = \begin{bmatrix} I & 0 \\ VA^{-1} & I \end{bmatrix} \begin{bmatrix} A & 0 \\ 0 & C - VA^{-1}U \end{bmatrix} \begin{bmatrix} I & A^{-1}U \\ 0 & I \end{bmatrix}</math>
:<math>\begin{bmatrix} A & U \\ V & C \end{bmatrix} = \begin{bmatrix} I & 0 \\ VA^{-1} & I \end{bmatrix} \begin{bmatrix} A & 0 \\ 0 & C - VA^{-1}U \end{bmatrix} \begin{bmatrix} I & A^{-1}U \\ 0 & I \end{bmatrix}</math>
जो ब्लॉक मैट्रिक्स का ऊपरी त्रिकोणीय, विकर्ण और निचले त्रिकोणीय मैट्रिक्स में एलडीयू अपघटन है।
मिलता है जो ब्लॉक आव्यूह का ऊपरी त्रिकोणीय, विकर्ण और निचले त्रिकोणीय आव्यूह में एलडीयू अपघटन है।


अब दोनों पक्षों को उलटने पर प्राप्त होता है
अब दोनों पक्षों को व्युत्क्रमित करने पर


: <math>\begin{align}
: <math>\begin{align}
Line 188: Line 191:
     &= \begin{bmatrix} I & -A^{-1}U \\ 0 & I \end{bmatrix} \begin{bmatrix} A^{-1} & 0 \\ 0 & \left(C - VA^{-1}U\right)^{-1} \end{bmatrix} \begin{bmatrix} I & 0 \\ -VA^{-1} & I \end{bmatrix} \\[8pt]
     &= \begin{bmatrix} I & -A^{-1}U \\ 0 & I \end{bmatrix} \begin{bmatrix} A^{-1} & 0 \\ 0 & \left(C - VA^{-1}U\right)^{-1} \end{bmatrix} \begin{bmatrix} I & 0 \\ -VA^{-1} & I \end{bmatrix} \\[8pt]
     &= \begin{bmatrix} A^{-1} + A^{-1}U\left(C - VA^{-1}U\right)^{-1}VA^{-1} & -A^{-1}U\left(C - VA^{-1}U\right)^{-1} \\ -\left(C - VA^{-1}U\right)^{-1}VA^{-1} & \left(C - VA^{-1}U\right)^{-1} \end{bmatrix}  \qquad\mathrm{(1)}
     &= \begin{bmatrix} A^{-1} + A^{-1}U\left(C - VA^{-1}U\right)^{-1}VA^{-1} & -A^{-1}U\left(C - VA^{-1}U\right)^{-1} \\ -\left(C - VA^{-1}U\right)^{-1}VA^{-1} & \left(C - VA^{-1}U\right)^{-1} \end{bmatrix}  \qquad\mathrm{(1)}
\end{align}</math>
\end{align}</math> प्राप्त होता है
हम इसे दूसरे तरीके से भी समान रूप से अच्छी तरह से कर सकते थे (बशर्ते कि C उलटा हो) यानी।
हम इसे समान रूप से दूसरी विधि से भी कर सकते थे (परंतु C व्युत्क्रम हो) अर्थात


: <math>\begin{bmatrix} A & U \\ V & C \end{bmatrix} = \begin{bmatrix} I & UC^{-1} \\ 0 & I \end{bmatrix} \begin{bmatrix} A - UC^{-1}V & 0 \\ 0 & C \end{bmatrix} \begin{bmatrix} I & 0 \\ C^{-1}V  & I\end{bmatrix}</math>
: <math>\begin{bmatrix} A & U \\ V & C \end{bmatrix} = \begin{bmatrix} I & UC^{-1} \\ 0 & I \end{bmatrix} \begin{bmatrix} A - UC^{-1}V & 0 \\ 0 & C \end{bmatrix} \begin{bmatrix} I & 0 \\ C^{-1}V  & I\end{bmatrix}</math>
अब फिर से दोनों पक्षों को उलटा करके,
अब फिर से दोनों पक्षों को व्युत्क्रमित करके,
: <math>\begin{align}
: <math>\begin{align}
   \begin{bmatrix} A & U \\ V & C \end{bmatrix}^{-1}
   \begin{bmatrix} A & U \\ V & C \end{bmatrix}^{-1}
Line 199: Line 202:
     &= \begin{bmatrix} \left(A - UC^{-1}V\right)^{-1} & -\left(A - UC^{-1}V\right)^{-1}UC^{-1} \\ -C^{-1}V\left(A - UC^{-1}V\right)^{-1} & C^{-1} + C^{-1}V\left(A - UC^{-1}V\right)^{-1}UC^{-1} \end{bmatrix} \qquad\mathrm{(2)}
     &= \begin{bmatrix} \left(A - UC^{-1}V\right)^{-1} & -\left(A - UC^{-1}V\right)^{-1}UC^{-1} \\ -C^{-1}V\left(A - UC^{-1}V\right)^{-1} & C^{-1} + C^{-1}V\left(A - UC^{-1}V\right)^{-1}UC^{-1} \end{bmatrix} \qquad\mathrm{(2)}
\end{align}</math>
\end{align}</math>
अब उपरोक्त (1) और (2) के आरएचएस के तत्वों (1, 1) की तुलना करने से वुडबरी फॉर्मूला मिलता है
अब उपरोक्त (1) और (2) के आरएचएस के अवयवों (1, 1) की तुलना करने से वुडबरी सूत्र
:<math>\left(A - UC^{-1}V\right)^{-1} = A^{-1} + A^{-1}U\left(C - VA^{-1}U\right)^{-1}VA^{-1}.</math>
:<math>\left(A - UC^{-1}V\right)^{-1} = A^{-1} + A^{-1}U\left(C - VA^{-1}U\right)^{-1}VA^{-1}</math>मिलता है।
{{collapse bottom}}
{{collapse bottom}}


== अनुप्रयोग ==
== अनुप्रयोग ==


यह समरूपता कुछ संख्यात्मक गणनाओं में उपयोगी है जहां <sup>−1</sup> की गणना पहले ही की जा चुकी है और (+यूसीवी) की गणना करना वांछित है<sup>−1</sup>A का व्युत्क्रम उपलब्ध होने पर, केवल C का व्युत्क्रम ज्ञात करना आवश्यक है<sup>−1 + वीए<sup>−1</sup>यू समरूपता के दाईं ओर का उपयोग करके परिणाम प्राप्त करने के लिए। यदि C का आयाम A से बहुत छोटा है, तो यह A + UCV को सीधे उलटने की तुलना में अधिक कुशल है। सामान्य मामला ए के निम्न-पद अपडेट ए + यूसीवी (जहां यू में केवल कुछ कॉलम हैं और वी में केवल कुछ पंक्तियां हैं) का व्युत्क्रम ढूंढना है, या आव्यूह + बी के व्युत्क्रम का अनुमान लगाना है जहां आव्यूह बी को निम्न-पद आव्यूह यूसीवी द्वारा अनुमानित किया जा सकता है, उदाहरण के लिए एकवचन मूल्य अपघटन का उपयोग करना।
अतः यह समरूपता कुछ संख्यात्मक गणनाओं में उपयोगी है जहां A<sup>−1</sup> की गणना पहले ही की जा चुकी है और (''A'' + ''UCV'')<sup>−1</sup> की गणना करना वांछित है। A का व्युत्क्रम उपलब्ध होने पर, पहचान के दाईं ओर का उपयोग करके परिणाम प्राप्त करने के लिए मात्र C<sup>−1 + ''VA''<sup>−1</sup>U का व्युत्क्रम ज्ञात करना आवश्यक है। यदि C की विमा A से बहुत छोटी है, तो यह A + UCV को प्रत्यक्षतः व्युत्क्रमित करने की तुलना में अधिक कुशल है। इस प्रकार से सामान्य स्थित A के निम्न-पद अद्यतन A + ''UCV'' (जहां ''U'' में मात्र कुछ स्तम्भ हैं और ''V'' में मात्र कुछ पंक्तियां हैं) का व्युत्क्रम ढूंढना है, या आव्यूह A + B के व्युत्क्रम का अनुमान लगाना है जहां आव्यूह B को निम्न-पद आव्यूह ''UCV'' द्वारा अनुमानित किया जा सकता है, उदाहरण के लिए एकल मान अपघटन का उपयोग करना।


इसे लागू किया जाता है, उदाहरण के लिए, [[कलमन फ़िल्टर]] और [[पुनरावर्ती न्यूनतम वर्ग]] विधियों में, [[पैरामीट्रिक समाधान|पैरामीट्रिक हल]] को बदलने के लिए, स्थिति समीकरण आधारित हल के साथ, राज्य सदिश आकार आव्यूह के व्युत्क्रम की आवश्यकता होती है। कलमन फ़िल्टर के स्थिति में इस आव्यूह में अवलोकनों के सदिश के आयाम होते हैं, यानी, समय में केवल नया अवलोकन संसाधित होने की स्थिति में 1 जितना छोटा होता है। यह फ़िल्टर की अक्सर वास्तविक समय गणनाओं को महत्वपूर्ण रूप से गति देता है।
इसे लागू किया जाता है, उदाहरण के लिए, [[कलमन फ़िल्टर|कलमन निस्पंदन]] और [[पुनरावर्ती न्यूनतम वर्ग]] विधियों में, [[पैरामीट्रिक समाधान|पैरामीट्रिक हल]] को बदलने के लिए, स्थिति समीकरण आधारित हल के साथ, अवस्था सदिश आकार आव्यूह के व्युत्क्रम की आवश्यकता होती है। इस प्रकार से कलमन निस्पंदन की स्थिति में इस आव्यूह में अवलोकनों की सदिश विमा होती है, अर्थात, समय में मात्र नवीन अवलोकन संसाधित होने की स्थिति में 1 जितना छोटा होता है। अतः यह निस्पंदन की प्रायः वास्तविक समय गणनाओं को महत्वपूर्ण रूप से गति देता है।


उस स्थिति में जब C समरूपता आव्यूह I है, आव्यूह <math>I+VA^{-1}U</math> [[संख्यात्मक रैखिक बीजगणित]] और [[संख्यात्मक आंशिक अंतर समीकरण]]ों में कैपेसिटेंस आव्यूह के रूप में जाना जाता है।<ref name="hager"/>
इस प्रकार से उस स्थिति में जब C समरूपता आव्यूह I है, आव्यूह <math>I+VA^{-1}U</math> को [[संख्यात्मक रैखिक बीजगणित]] और [[संख्यात्मक आंशिक अंतर समीकरण|संख्यात्मक आंशिक अंतर समीकरणों]] में '''संधारित्र आव्यूह''' के रूप में जाना जाता है।<ref name="hager"/>
==यह भी देखें==
==यह भी देखें==
*शर्मन-मॉरिसन सूत्र
*शर्मन-मॉरिसन सूत्र
*[[शूर पूरक]]
*[[शूर पूरक]]
*[[मैट्रिक्स निर्धारक लेम्मा|आव्यूह निर्धारक लेम्मा]], निर्धारक के लिए पद-k अद्यतन के लिए सूत्र
*[[मैट्रिक्स निर्धारक लेम्मा|आव्यूह निर्धारक लेम्मा]], निर्धारक के लिए पद-k अद्यतन के लिए सूत्र
*[[उलटा मैट्रिक्स|उलटा आव्यूह]]
*[[उलटा मैट्रिक्स|व्युत्क्रम आव्यूह]]
*मूर-पेनरोज़ स्यूडोइनवर्स#स्यूडोइनवर्स को अद्यतन कर रहा है
*मूर-पेनरोज़ छद्मइ[[उलटा मैट्रिक्स|व्युत्क्रम]] छद्म[[उलटा मैट्रिक्स|व्युत्क्रम]] को अद्यतन कर रहा है


== टिप्पणियाँ ==
== टिप्पणियाँ ==

Revision as of 21:20, 22 July 2023

गणित में (विशेष रूप से रैखिक बीजगणित), वुडबरी आव्यूह समरूपता, जिसका नाम मैक्स ए. वुडबरी के नाम पर रखा गया है,[1][2] जो यह कहते है कि कुछ आव्यूह (गणित) के पद-k सुधार के व्युत्क्रम की गणना मूल आव्यूह के व्युत्क्रम में पद-k सुधार करके की जा सकती है। अतः इस सूत्र के वैकल्पिक नाम 'आव्यूह व्युत्क्रमता लेम्मा', 'शर्मन-मॉरिसन-वुडबरी सूत्र' या मात्र 'वुडबरी सूत्र' हैं। यद्यपि, वुडबरी रिपोर्ट से पहले यह समरूपता कई लेखों में छपी थी।[3][4]

इस प्रकार से वुडबरी आव्यूह समरूपता[5]

है, जहां A, U, C और V अनुरूप आव्यूह हैं: A n×n है, C k×k है, U n×k है, और V k×n है। इसे व्युत्क्रमणीय आव्यूह ब्लॉक वार व्युत्क्रम का उपयोग करके प्राप्त किया जा सकता है।

जबकि समरूपता मुख्य रूप से आव्यूह पर उपयोग की जाती है, यह सामान्य वलय (गणित) या Ab-श्रेणी में होती है।

इस प्रकार से वुडबरी आव्यूह समरूपता व्युत्क्रमों की तुच्छ गणना और रैखिक समीकरणों के हल की अनुमति देती है। यद्यपि, सूत्र की संख्यात्मक स्थिरता के विषय में बहुत कम सूचना है। अतः इसकी त्रुटि सीमा के संबंध में कोई प्रकाशित परिणाम नहीं हैं। उपाख्यानात्मक प्रमाण [6] से पता चलता है कि यह प्रतीत होने वाले सौम्य उदाहरणों के लिए भी भिन्न हो सकता है (जब मूल और संशोधित आव्यूह दोनों ठीक रूप से प्रतिबंधित हैं)।

चर्चा

इस परिणाम को सिद्ध करने के लिए, हम सरल परिणाम को सिद्ध करके प्रारम्भ करेंगे। इस प्रकार से A और C को समरूपता आव्यूह I के साथ प्रतिस्थापित करने पर, हमें और समरूपता प्राप्त होती है जो थोड़ी सरल है:

अतः इस घटी हुई समरूपता से मूल समीकरण को पुनः प्राप्त करने के लिए, समुच्चय और है।

इस समरूपता को ही दो सरल समरूपताों के संयोजन के रूप में देखा जा सकता है। इस प्रकार से हम पहली समरूपता

से प्राप्त करते हैं, इस प्रकार,

,

और इसी प्रकार

दूसरी समरूपता तथाकथित पुश-थ्रू समरूपता[7]

है जिसे हम दाईं ओर और बाईं ओर

से गुणा करने के बाद

से प्राप्त करते हैं।

सभी को एक साथ रखने पर,

जहां पहली और दूसरी समानता क्रमशः पहली और दूसरी समरूपता से आती है।

विशेष स्थिति

जब सदिश होते हैं, तो समरूपता शर्मन-मॉरिसन सूत्र तक कम हो जाती है।

इस प्रकार से अदिश स्थिति में, घटा हुआ संस्करण मात्र

है।

योग का व्युत्क्रम

यदि n = k और U = V = In तो, समरूपता आव्यूह

है।

उपरोक्त समीकरण के सबसे दाईं ओर के पदों के एक संविलय को जारी रखने से हुआ की समरूपता

प्राप्त होती है।

इस प्रकार से समान समरूपता का एक अन्य उपयोगी रूप

है, जो उपरोक्त के विपरीत, एकल होने पर भी मान्य है, और इसमें एक पुनरावर्ती संरचना है जो का वर्णक्रमीय त्रिज्या एक से कम होने पर

उत्पन्न करती है। अर्थात्, यदि उपरोक्त योग अभिसरित होता है तो यह के बराबर होता है।

अतः इस रूप का उपयोग त्रुटि वाले विस्तारों में किया जा सकता है जहां B A की त्रुटि है।

विविधताएँ

द्विपद व्युत्क्रम प्रमेय

यदि A, B, U, V क्रमशः n×n, k×k, n×k, k×n आकार के आव्यूह हैं, तो

प्रदान किया गया है कि A और B + BVA−1UB एकल नहीं हैं। इस प्रकार से उत्तरार्द्ध की गैर-विलक्षणता के लिए आवश्यक है कि B−1 अस्तित्व में हो क्योंकि यह B(I + VA−1UB) के बराबर है और बाद वाले की पद B की पद से अधिक नहीं हो सकती है।[7]

चूँकि B व्युत्क्रमणीय है, दाहिनी ओर कोष्ठक में व्युत्क्रमित मात्रा को दर्शाने वाले दो B पदों को (B−1)−1 से प्रतिस्थापित किया जा सकता है, जिसके परिणामस्वरूप मूल वुडबरी समरूपता प्राप्त होती है।

इस प्रकार से जब B एकल हो और संभवतः गैर-वर्ग भी हो, तो इसके लिए भिन्नता:[7]

अतः कुछ स्थितियों के लिए सूत्र भी स्थित हैं जिनमें A एकल है।[8]

धनात्मक अर्धनिश्चित आव्यूह के साथ छद्म व्युत्क्रम

इस प्रकार से सामान्यतः वुडबरी की समरूपता मान्य नहीं है यदि या अधिक व्युत्क्रमों को मूर-पेनरोज़ व्युत्क्रम या (मूर-पेनरोज़) छद्म व्युत्क्रम द्वारा प्रतिस्थापित किया जाता है। यद्यपि, यदि और धनात्मक अर्धनिश्चित आव्यूह हैं, और (इसका तात्पर्य यह है कि स्वयं धनात्मक अर्धनिश्चित है), तो निम्न सूत्र सामान्यीकरण प्रदान करता है:[9][10]

जहाँ को के रूप में लिखा जा सकता है क्योंकि कोई भी धनात्मक अर्धनिश्चित आव्यूह कुछ के लिए के बराबर है।

व्युत्पत्तियाँ

प्रत्यक्ष प्रमाण

इस प्रकार से सूत्र को यह जांच कर सिद्ध किया जा सकता है कि वुडबरी समरूपता के दाईं ओर गुना इसके कथित व्युत्क्रम से समरूपता आव्यूह मिलता है: